Please confirm topic selection

Are you sure you want to trigger topic in your Anconeus AI algorithm?

Please confirm action

You are done for today with this topic.

Would you like to start learning session with this topic items scheduled for future?

Review Question - QID 215179

In scope icon M 2 A
QID 215179 (Type "215179" in App Search)
A 70-year-old man presents to the emergency department with worsening shortness of breath. He used to walk 2-3 miles every morning but over the past few months, his exercise tolerance has diminished. Now, he cannot take more than 10 steps without sitting to catch his breath. He has a history of diabetes, hypertension, and triple coronary bypass surgery. He has a 50 pack-year smoking history and drinks 6-7 beers per week. He has no recent travel history. The patient’s temperature is 98.2°F (36.8°C), blood pressure is 154/92 mmHg, pulse is 78/min, and respirations are 16/min. Physical exam is notable for 2+ pitting edema bilaterally, jugular venous pulsations 6 cm above the sternal notch, and an S3 murmur. An echocardiogram shows a left ventricular ejection fraction of 40%. What are the expected changes in total peripheral resistance, renin, angiotensin II, aldosterone, and bicarbonate (HCO3-) in this patient (Figure A)?
  • A
  • A